IASbaba Prelims 60 Days Plan, Rapid Revision Series (RaRe)
Archives
Hello Friends
The 60 Days Rapid Revision (RaRe) Series is IASbaba’s Flagship Initiative recommended by Toppers and loved by the aspirants’ community every year.
It is the most comprehensive program which will help you complete the syllabus, revise and practice tests on a daily basis. The Programme on a daily basis includes
Daily Prelims MCQs from Static (Monday – Saturday)
- Daily Static Quiz will cover all the topics of static subjects – Polity, History, Geography, Economics, Environment and Science and technology.
- 20 questions will be posted daily and these questions are framed from the topics mentioned in the schedule.
- It will ensure timely and streamlined revision of your static subjects.
Daily Current Affairs MCQs (Monday – Saturday)
- Daily 5 Current Affairs questions, based on sources like ‘The Hindu’, ‘Indian Express’ and ‘PIB’, would be published from Monday to Saturday according to the schedule.
Daily CSAT Quiz (Monday – Friday)
- CSAT has been an Achilles heel for many aspirants.
- Daily 5 CSAT Questions will be published.
Note – Daily Test of 20 static questions, 10 current affairs, and 5 CSAT questions. (35 Prelims Questions) in QUIZ FORMAT will be updated on a daily basis.
To Know More about 60 Days Rapid Revision (RaRe) Series – CLICK HERE
60 Day Rapid Revision (RaRe) Series Schedule – CLICK HERE
Important Note
- Comment your Scores in the Comment Section. This will keep you accountable, responsible and sincere in days to come.
- It will help us come out with the Cut-Off on a Daily Basis.
- Let us know if you enjoyed today’s test 🙂
- You can post your comments in the given format
- (1) Your Score
- (2) Matrix Meter
- (3) New Learning from the Test
Test-summary
0 of 35 questions completed
Questions:
- 1
- 2
- 3
- 4
- 5
- 6
- 7
- 8
- 9
- 10
- 11
- 12
- 13
- 14
- 15
- 16
- 17
- 18
- 19
- 20
- 21
- 22
- 23
- 24
- 25
- 26
- 27
- 28
- 29
- 30
- 31
- 32
- 33
- 34
- 35
Information
The following Test is based on the syllabus of 60 Days Plan-2023 for UPSC IAS Prelims 2022.
To view Solutions, follow these instructions:
- Click on – ‘Start Test’ button
- Solve Questions
- Click on ‘Test Summary’ button
- Click on ‘Finish Test’ button
- Now click on ‘View Questions’ button – here you will see solutions and links.
You have already completed the test before. Hence you can not start it again.
Test is loading...
You must sign in or sign up to start the test.
You have to finish following test, to start this test:
Results
0 of 35 questions answered correctly
Your time:
Time has elapsed
You have scored 0 points out of 0 points, (0)
Average score |
|
Your score |
|
Categories
- Not categorized 0%
Pos. | Name | Entered on | Points | Result |
---|---|---|---|---|
Table is loading | ||||
No data available | ||||
- 1
- 2
- 3
- 4
- 5
- 6
- 7
- 8
- 9
- 10
- 11
- 12
- 13
- 14
- 15
- 16
- 17
- 18
- 19
- 20
- 21
- 22
- 23
- 24
- 25
- 26
- 27
- 28
- 29
- 30
- 31
- 32
- 33
- 34
- 35
- Answered
- Review
-
Question 1 of 35
1. Question
In which of the following matters, the State Legislative Council of a particular state is not equal with the State Legislative Assembly of that particular state?
- Election of the President of India
- Elections to the Rajya Sabha
- Ratification of the Constitutional Amendment
- Introduction of Confidence Motion
Select the correct answer using the code given below:
Correct
Solution (d)
Statement 1 Statement 2 Statement 3 Statement 4 Correct Correct Correct Correct The elected members of state legislative assemblies participate in the elections of the President whereas the council does not. The elected members of state legislative assemblies participate in the elections of the representatives of the states in the Rajya Sabha whereas the council does not. The council has no effective say in the ratification of a constitutional amendment bill. In this respect, the will of the assembly prevails over that of the council.
The council cannot remove the council of ministers by passing a no-confidence motion. This is because, the council of ministers is collectively responsible only to the assembly.
But, the council can discus and criticise the policies and activities of the Government.
Note:
POSITION OF LEGISLATIVE COUNCIL:
The constitutional position of the council (as compared with the assembly) can be studied from two angles:
- Spheres where council is equal to assembly.
- Spheres where council is unequal to assembly.
Equal with Assembly:
In the following matters, the powers and status of the council are broadly equal to that of the assembly:
- Introduction and passage of ordinary bills. However, in case of disagreement between the two Houses, the will of the assembly prevails over that of the council.
- Approval of ordinances issued by the governor.
- Selection of ministers including the chief minister. Under the Constitution the, ministers including the chief minister can be members of either House of the state legislature. However, irrespective of their membership, they are responsible only to the assembly.
- Consideration of the reports of the constitutional bodies like State Finance Commission, state public service commission and Comptroller and Auditor General of India.
- Enlargement of the jurisdiction of the state public service commission.
Unequal with Assembly:
In the following matters, the powers and status of the council are unequal to that of the assembly:
- A Money Bill can be introduced only in the assembly and not in the council.
- The council cannot amend or reject a money bill. It should return the bill to the assembly within 14 days, either with recommendations or without recommendations.
- The assembly can either accept or reject all or any of the recommendation of the council. In both the cases, the money bill is deemed to have been passed by the two Houses.
- The final power to decide whether a particular bill is a money bill or not is vested in the Speaker of the assembly.
- The final power of passing an ordinary bill also lies with the assembly. At the most, the council can detain or delay the bill for the period of four months–three months in the first instance and one month in the second instance. In other words, the council is not even a revising body like the Rajya Sabha; it is only a dilatory chamber or an advisory body.
- The council can only discuss the budget but cannot vote on the demands for grants (which is the exclusive privilege of the assembly).
- The council cannot remove the council of ministers by passing a no-confidence motion. This is because, the council of ministers is collectively responsible only to the assembly. But, the council can discus and criticise the policies and activities of the Government.
- When an ordinary bill, which has originated in the council and was sent to the assembly, is rejected by the assembly, the bill ends and becomes dead.
- The council does not participate in the election of the president of India and representatives of the state in the Rajya Sabha.
- The council has no effective say in the ratification of a constitutional amendment bill. In this respect also, the will of the assembly prevails over that of the council
- Finally, the very existence of the council depends on the will of the assembly. The council can be abolished by the Parliament on the recommendation of the assembly.
From the above, it is clear that the position of the council vis-a-vis the assembly is much weaker than the position of the Rajya Sabha vis-a-vis the Lok Sabha. The Rajya Sabha has equal powers with the Lok Sabha in all spheres except financial matters and with regard to the control over the Government. On the other hand, the council is subordinate to the assembly in all respects. Thus, the predominance of the assembly over the council is fully established.
Incorrect
Solution (d)
Statement 1 Statement 2 Statement 3 Statement 4 Correct Correct Correct Correct The elected members of state legislative assemblies participate in the elections of the President whereas the council does not. The elected members of state legislative assemblies participate in the elections of the representatives of the states in the Rajya Sabha whereas the council does not. The council has no effective say in the ratification of a constitutional amendment bill. In this respect, the will of the assembly prevails over that of the council.
The council cannot remove the council of ministers by passing a no-confidence motion. This is because, the council of ministers is collectively responsible only to the assembly.
But, the council can discus and criticise the policies and activities of the Government.
Note:
POSITION OF LEGISLATIVE COUNCIL:
The constitutional position of the council (as compared with the assembly) can be studied from two angles:
- Spheres where council is equal to assembly.
- Spheres where council is unequal to assembly.
Equal with Assembly:
In the following matters, the powers and status of the council are broadly equal to that of the assembly:
- Introduction and passage of ordinary bills. However, in case of disagreement between the two Houses, the will of the assembly prevails over that of the council.
- Approval of ordinances issued by the governor.
- Selection of ministers including the chief minister. Under the Constitution the, ministers including the chief minister can be members of either House of the state legislature. However, irrespective of their membership, they are responsible only to the assembly.
- Consideration of the reports of the constitutional bodies like State Finance Commission, state public service commission and Comptroller and Auditor General of India.
- Enlargement of the jurisdiction of the state public service commission.
Unequal with Assembly:
In the following matters, the powers and status of the council are unequal to that of the assembly:
- A Money Bill can be introduced only in the assembly and not in the council.
- The council cannot amend or reject a money bill. It should return the bill to the assembly within 14 days, either with recommendations or without recommendations.
- The assembly can either accept or reject all or any of the recommendation of the council. In both the cases, the money bill is deemed to have been passed by the two Houses.
- The final power to decide whether a particular bill is a money bill or not is vested in the Speaker of the assembly.
- The final power of passing an ordinary bill also lies with the assembly. At the most, the council can detain or delay the bill for the period of four months–three months in the first instance and one month in the second instance. In other words, the council is not even a revising body like the Rajya Sabha; it is only a dilatory chamber or an advisory body.
- The council can only discuss the budget but cannot vote on the demands for grants (which is the exclusive privilege of the assembly).
- The council cannot remove the council of ministers by passing a no-confidence motion. This is because, the council of ministers is collectively responsible only to the assembly. But, the council can discus and criticise the policies and activities of the Government.
- When an ordinary bill, which has originated in the council and was sent to the assembly, is rejected by the assembly, the bill ends and becomes dead.
- The council does not participate in the election of the president of India and representatives of the state in the Rajya Sabha.
- The council has no effective say in the ratification of a constitutional amendment bill. In this respect also, the will of the assembly prevails over that of the council
- Finally, the very existence of the council depends on the will of the assembly. The council can be abolished by the Parliament on the recommendation of the assembly.
From the above, it is clear that the position of the council vis-a-vis the assembly is much weaker than the position of the Rajya Sabha vis-a-vis the Lok Sabha. The Rajya Sabha has equal powers with the Lok Sabha in all spheres except financial matters and with regard to the control over the Government. On the other hand, the council is subordinate to the assembly in all respects. Thus, the predominance of the assembly over the council is fully established.
-
Question 2 of 35
2. Question
Consider the following statements about the Legislative Council:
- A special resolution is passed for both the creation and the abolishment of the Legislative Council by the State Assembly.
- All the members of the Legislative Council are indirectly elected by different constituents.
- The Parliament can change the composition of the Legislative Council.
- The Constitution has placed the maximum and the minimum membership of the Legislative Councils.
How many of the above statements are correct?
Correct
Solution (c)
Statement 1 Statement 2 Statement 3 Statement 4 Correct Incorrect Correct Correct Notwithstanding anything in Article 168, the Parliament may, by law, provide for the abolition of the Legislative Council of a state having such a Council or for the creation of such a Council in a state having no such Council, if the Legislative Assembly of the state passes a resolution to that effect by a majority of the total membership of the Assembly and by a majority of not less than two-thirds of the members of the Assembly present and voting. Some members of the Legislative Council are indirectly elected while some are nominated by the Governor. 5/6 of the total number of members of a legislative council are indirectly
elected and 1/6 are nominated by the governor.
The Parliament by law can change the composition of the Legislative Council. However, the Parliament has not enacted a law to that effect.
The total number of the members in the Legislative Council of a state, having such a Council, shall not exceed one-third of the total number of the members in the Legislative Assembly of that state: Provided that the total number of the members in the Legislative Council of a state shall in no case be less than 40. Incorrect
Solution (c)
Statement 1 Statement 2 Statement 3 Statement 4 Correct Incorrect Correct Correct Notwithstanding anything in Article 168, the Parliament may, by law, provide for the abolition of the Legislative Council of a state having such a Council or for the creation of such a Council in a state having no such Council, if the Legislative Assembly of the state passes a resolution to that effect by a majority of the total membership of the Assembly and by a majority of not less than two-thirds of the members of the Assembly present and voting. Some members of the Legislative Council are indirectly elected while some are nominated by the Governor. 5/6 of the total number of members of a legislative council are indirectly
elected and 1/6 are nominated by the governor.
The Parliament by law can change the composition of the Legislative Council. However, the Parliament has not enacted a law to that effect.
The total number of the members in the Legislative Council of a state, having such a Council, shall not exceed one-third of the total number of the members in the Legislative Assembly of that state: Provided that the total number of the members in the Legislative Council of a state shall in no case be less than 40. -
Question 3 of 35
3. Question
According to the provisions of the Constitution of India, which of the following qualifications are mandatory to be chosen as a member of the state legislature?
- Minimum age of 30 years in case of the Legislative Assembly and 35 years in case of the Legislative Council.
- An elector for the assembly constituency in the concerned state.
- A resident in the concerned state to be eligible for governor’s nomination.
How many of the above statements are correct?
Correct
Solution (d)
Statement 1 Statement 2 Statement 3 Incorrect Incorrect Incorrect The state legislative members, he/she must be not less than 25 years of age in the case of legislative assembly and must be not less than 30 years of age in case of legislative council. Qualification for a member of state legislature to be an elector for an assembly constituency in the same state is not constitutional provisions. It is an additional qualification laid down by the Parliament in the Representation of People Act (1951).
Qualification for a member of state legislature to be a resident in the concerned state to be eligible for Governor’s nomination is not constitutional provisions. It is an additional qualification laid down by the Parliament in the Representation of People Act (1951).
Note:
Qualifications for the membership of the State Legislature:
The Constitution lays down the following qualification for a person to be chosen a member of the state legislature:
- He must be a citizen of India.
- He must make and subscribe to an oath or affirmation before the person authorized by the Election commission for this purpose. In his oath or affirmation, he swears
- To bear true faith and allegiance to the Constitution of India.
- To uphold the sovereignty and integrity of India.
- He must be not less than 25 years of age in case of legislative assembly and not less than 30 in case of legislative council.
- He must possess other qualifications prescribed by parliament.
Accordingly, the parliament has laid down the following additional qualifications in the Representation of peoples act 1951:
- A person to be elected to the legislative council must be an elector for an assembly constituency in the concerned state and to be qualified for the Governor’s nomination, he must be a resident in the concerned state.
- A person to be elected to the legislative assembly must be an elector for an assembly constituency in the concerned state.
- He must be a member of a scheduled caste or scheduled tribe if he wants to contest a seat reserved for them. Also, they can contest a seat not reserved for them.
Incorrect
Solution (d)
Statement 1 Statement 2 Statement 3 Incorrect Incorrect Incorrect The state legislative members, he/she must be not less than 25 years of age in the case of legislative assembly and must be not less than 30 years of age in case of legislative council. Qualification for a member of state legislature to be an elector for an assembly constituency in the same state is not constitutional provisions. It is an additional qualification laid down by the Parliament in the Representation of People Act (1951).
Qualification for a member of state legislature to be a resident in the concerned state to be eligible for Governor’s nomination is not constitutional provisions. It is an additional qualification laid down by the Parliament in the Representation of People Act (1951).
Note:
Qualifications for the membership of the State Legislature:
The Constitution lays down the following qualification for a person to be chosen a member of the state legislature:
- He must be a citizen of India.
- He must make and subscribe to an oath or affirmation before the person authorized by the Election commission for this purpose. In his oath or affirmation, he swears
- To bear true faith and allegiance to the Constitution of India.
- To uphold the sovereignty and integrity of India.
- He must be not less than 25 years of age in case of legislative assembly and not less than 30 in case of legislative council.
- He must possess other qualifications prescribed by parliament.
Accordingly, the parliament has laid down the following additional qualifications in the Representation of peoples act 1951:
- A person to be elected to the legislative council must be an elector for an assembly constituency in the concerned state and to be qualified for the Governor’s nomination, he must be a resident in the concerned state.
- A person to be elected to the legislative assembly must be an elector for an assembly constituency in the concerned state.
- He must be a member of a scheduled caste or scheduled tribe if he wants to contest a seat reserved for them. Also, they can contest a seat not reserved for them.
-
Question 4 of 35
4. Question
Consider the following statements about the Speaker of the Legislative Assembly:
- He decides whether a Bill is a Money Bill or not, and his decision on this question is final.
- He decides the question of the disqualification of a member of the Assembly, arising on the ground of defection.
- He is the final interpreter of the provisions of the Constitution of India within the Assembly.
How many of the above statements are correct?
Correct
Solution (c)
Statement 1 Statement 2 Statement 3 Correct Correct Correct The Speaker of Legislative Assembly decides whether a Bill is a Money Bill or not, and his decision on this question is final. The Speaker of Legislative Assembly decides the question of the disqualification of a member of the Assembly, arising on the ground of defection. The Speaker of Legislative Assembly is the final interpreter of the provisions of the Constitution of India within the Assembly. The Speaker of Legislative Assembly has the following powers and duties –
- He maintains order and decorum in the Assembly for conducting its business and regulating its proceedings. This is his primary responsibility and he has the final power in this regard.
- He is the final interpreter of the provisions of (a) the Constitution of India; (b) the rules of procedure and conduct of the business of the Assembly; and (c) the legislative precedents, within the Assembly.
- He adjourns the Assembly or suspends the meeting in the absence of a quorum.
- He does not vote in the first instance. But he can exercise a casting vote in the case of a tie
- He can allow a ‘secret’ sitting of the House at the request of the Leader of the House.
- He decides whether a Bill is a Money Bill or not, and his decision on this question is final.
- He decides the question of the disqualification of a member of the Assembly, arising on the ground of defection, under the provisions of the Tenth Schedule.
- He appoints the Chairmen of all the Committees of the Assembly and supervises their functioning. He himself is the Chairman of the Business Advisory Committee; the Rules Committee; and the General-Purpose Committee.
Incorrect
Solution (c)
Statement 1 Statement 2 Statement 3 Correct Correct Correct The Speaker of Legislative Assembly decides whether a Bill is a Money Bill or not, and his decision on this question is final. The Speaker of Legislative Assembly decides the question of the disqualification of a member of the Assembly, arising on the ground of defection. The Speaker of Legislative Assembly is the final interpreter of the provisions of the Constitution of India within the Assembly. The Speaker of Legislative Assembly has the following powers and duties –
- He maintains order and decorum in the Assembly for conducting its business and regulating its proceedings. This is his primary responsibility and he has the final power in this regard.
- He is the final interpreter of the provisions of (a) the Constitution of India; (b) the rules of procedure and conduct of the business of the Assembly; and (c) the legislative precedents, within the Assembly.
- He adjourns the Assembly or suspends the meeting in the absence of a quorum.
- He does not vote in the first instance. But he can exercise a casting vote in the case of a tie
- He can allow a ‘secret’ sitting of the House at the request of the Leader of the House.
- He decides whether a Bill is a Money Bill or not, and his decision on this question is final.
- He decides the question of the disqualification of a member of the Assembly, arising on the ground of defection, under the provisions of the Tenth Schedule.
- He appoints the Chairmen of all the Committees of the Assembly and supervises their functioning. He himself is the Chairman of the Business Advisory Committee; the Rules Committee; and the General-Purpose Committee.
-
Question 5 of 35
5. Question
In the context of Legislative procedure in State Legislature with regard to Money Bill, consider the following statements:
- Money bill in state legislature can be introduced in either Legislative assembly or Legislative council.
- Legislative council cannot reject money bill and can only amend it.
- Governor can reserve the money bill for presidential assent.
- Money bill cannot be returned for reconsideration of the state legislature either by the Governor or by the President.
How many of the above statements are correct?
Correct
Solution (b)
Statement 1 Statement 2 Statement 3 Statement 4 Incorrect Incorrect Correct Correct Money Bill cannot be introduced in the legislative council. It can be introduced in the legislative assembly only and that too on the recommendation of the governor. The legislative council has restricted powers with regard to a Money Bill. It cannot reject or amend a Money Bill. It can only make recommendations and must return the bill to the legislative assembly within 14 days. The legislative assembly can either accept or reject all or any of the recommendations of the legislative council. When a Money Bill is presented to the governor, he may either give his assent, withhold his assent or reserve the bill for presidential assent. Money bill cannot be returned for reconsideration of the state legislature either by the Governor. Also, when a money bill is reserved for consideration of the President, the president may either give his assent to the bill or withhold his assent to the bill but cannot return the bill for reconsideration of the state legislature.
Incorrect
Solution (b)
Statement 1 Statement 2 Statement 3 Statement 4 Incorrect Incorrect Correct Correct Money Bill cannot be introduced in the legislative council. It can be introduced in the legislative assembly only and that too on the recommendation of the governor. The legislative council has restricted powers with regard to a Money Bill. It cannot reject or amend a Money Bill. It can only make recommendations and must return the bill to the legislative assembly within 14 days. The legislative assembly can either accept or reject all or any of the recommendations of the legislative council. When a Money Bill is presented to the governor, he may either give his assent, withhold his assent or reserve the bill for presidential assent. Money bill cannot be returned for reconsideration of the state legislature either by the Governor. Also, when a money bill is reserved for consideration of the President, the president may either give his assent to the bill or withhold his assent to the bill but cannot return the bill for reconsideration of the state legislature.
-
Question 6 of 35
6. Question
Consider the following statements regarding Composition of Legislative Council:
- The size of the council depends on the size of the assembly of the concerned state.
- 5/6 of the total number of members of a legislative council are directly elected and 1/6 are nominated by the governor.
- The scheme of composition of a legislative council as laid down in the Constitution is tentative and not final.
How many of the above statements are correct?
Correct
Solution (b)
Statement 1 Statement 2 Statement 3 Correct Incorrect Correct The maximum strength of the council is fixed at one-third of the total strength of the assembly and the minimum strength is fixed at 40. It means that the size of the council depends on the size of the assembly of the concerned state. 5/6 of the total number of members of a legislative council are indirectly elected and 1/6 are nominated by the governor. The members are elected in accordance with the system of proportional representation by means of a single transferable vote. This scheme of composition of a legislative council as laid down in the Constitution is tentative and not final. The Parliament is authorized to modify or replace the same. However, it has not enacted any such law so far. Incorrect
Solution (b)
Statement 1 Statement 2 Statement 3 Correct Incorrect Correct The maximum strength of the council is fixed at one-third of the total strength of the assembly and the minimum strength is fixed at 40. It means that the size of the council depends on the size of the assembly of the concerned state. 5/6 of the total number of members of a legislative council are indirectly elected and 1/6 are nominated by the governor. The members are elected in accordance with the system of proportional representation by means of a single transferable vote. This scheme of composition of a legislative council as laid down in the Constitution is tentative and not final. The Parliament is authorized to modify or replace the same. However, it has not enacted any such law so far. -
Question 7 of 35
7. Question
Consider the following statements regarding the legislative procedure in the State Legislature:
- There is a system of joint sitting in state legislature to resolve the deadlock between the two Houses on passing the Bills.
- If an ordinary Bill passed by the Council is transmitted to the Assembly and rejected by the latter, there is an end to the Bill.
Which of the statements given above is/are correct?
Correct
Solution (b)
Statement 1 Statement 2 Incorrect Correct The Constitution does not provide for the mechanism of joint sitting of both the Houses in the State to resolve the disagreement between the two Houses over a bill. When an ordinary bill, which has originated in the council and was sent to the assembly, is rejected by the assembly, the bill ends and becomes dead. Note:
POSITION OF LEGISLATIVE COUNCIL:
The constitutional position of the council (as compared with the assembly) can be studied from two angles:
- Spheres where council is equal to assembly.
- Spheres where council is unequal to assembly.
Equal with Assembly:
In the following matters, the powers and status of the council are broadly equal to that of the assembly:
- Introduction and passage of ordinary bills. However, in case of disagreement between the two Houses, the will of the assembly prevails over that of the council.
- Approval of ordinances issued by the governor.
- Selection of ministers including the chief minister. Under the Constitution the, ministers including the chief minister can be members of either House of the state legislature. However, irrespective of their membership, they are responsible only to the assembly.
- Consideration of the reports of the constitutional bodies like State Finance Commission, state public service commission and Comptroller and Auditor General of India.
- Enlargement of the jurisdiction of the state public service commission.
Unequal with Assembly:
In the following matters, the powers and status of the council are unequal to that of the assembly:
- A Money Bill can be introduced only in the assembly and not in the council.
- The council cannot amend or reject a money bill. It should return the bill to the assembly within 14 days, either with recommendations or without recommendations.
- The assembly can either accept or reject all or any of the recommendation of the council. In both the cases, the money bill is deemed to have been passed by the two Houses.
- The final power to decide whether a particular bill is a money bill or not is vested in the Speaker of the assembly.
- The final power of passing an ordinary bill also lies with the assembly. At the most, the council can detain or delay the bill for the period of four months–three months in the first instance and one month in the second instance. In other words, the council is not even a revising body like the Rajya Sabha; it is only a dilatory chamber or an advisory body.
- The council can only discuss the budget but cannot vote on the demands for grants (which is the exclusive privilege of the assembly).
- The council cannot remove the council of ministers by passing a no-confidence motion. This is because, the council of ministers is collectively responsible only to the assembly. But, the council can discus and criticise the policies and activities of the Government.
- When an ordinary bill, which has originated in the council and was sent to the assembly, is rejected by the assembly, the bill ends and becomes dead.
- The council does not participate in the election of the president of India and representatives of the state in the Rajya Sabha.
- The council has no effective say in the ratification of a constitutional amendment bill. In this respect also, the will of the assembly prevails over that of the council
- Finally, the very existence of the council depends on the will of the assembly. The council can be abolished by the Parliament on the recommendation of the assembly.
From the above, it is clear that the position of the council vis-a-vis the assembly is much weaker than the position of the Rajya Sabha vis-a-vis the Lok Sabha. The Rajya Sabha has equal powers with the Lok Sabha in all spheres except financial matters and with regard to the control over the Government. On the other hand, the council is subordinate to the assembly in all respects. Thus, the predominance of the assembly over the council is fully established.
Incorrect
Solution (b)
Statement 1 Statement 2 Incorrect Correct The Constitution does not provide for the mechanism of joint sitting of both the Houses in the State to resolve the disagreement between the two Houses over a bill. When an ordinary bill, which has originated in the council and was sent to the assembly, is rejected by the assembly, the bill ends and becomes dead. Note:
POSITION OF LEGISLATIVE COUNCIL:
The constitutional position of the council (as compared with the assembly) can be studied from two angles:
- Spheres where council is equal to assembly.
- Spheres where council is unequal to assembly.
Equal with Assembly:
In the following matters, the powers and status of the council are broadly equal to that of the assembly:
- Introduction and passage of ordinary bills. However, in case of disagreement between the two Houses, the will of the assembly prevails over that of the council.
- Approval of ordinances issued by the governor.
- Selection of ministers including the chief minister. Under the Constitution the, ministers including the chief minister can be members of either House of the state legislature. However, irrespective of their membership, they are responsible only to the assembly.
- Consideration of the reports of the constitutional bodies like State Finance Commission, state public service commission and Comptroller and Auditor General of India.
- Enlargement of the jurisdiction of the state public service commission.
Unequal with Assembly:
In the following matters, the powers and status of the council are unequal to that of the assembly:
- A Money Bill can be introduced only in the assembly and not in the council.
- The council cannot amend or reject a money bill. It should return the bill to the assembly within 14 days, either with recommendations or without recommendations.
- The assembly can either accept or reject all or any of the recommendation of the council. In both the cases, the money bill is deemed to have been passed by the two Houses.
- The final power to decide whether a particular bill is a money bill or not is vested in the Speaker of the assembly.
- The final power of passing an ordinary bill also lies with the assembly. At the most, the council can detain or delay the bill for the period of four months–three months in the first instance and one month in the second instance. In other words, the council is not even a revising body like the Rajya Sabha; it is only a dilatory chamber or an advisory body.
- The council can only discuss the budget but cannot vote on the demands for grants (which is the exclusive privilege of the assembly).
- The council cannot remove the council of ministers by passing a no-confidence motion. This is because, the council of ministers is collectively responsible only to the assembly. But, the council can discus and criticise the policies and activities of the Government.
- When an ordinary bill, which has originated in the council and was sent to the assembly, is rejected by the assembly, the bill ends and becomes dead.
- The council does not participate in the election of the president of India and representatives of the state in the Rajya Sabha.
- The council has no effective say in the ratification of a constitutional amendment bill. In this respect also, the will of the assembly prevails over that of the council
- Finally, the very existence of the council depends on the will of the assembly. The council can be abolished by the Parliament on the recommendation of the assembly.
From the above, it is clear that the position of the council vis-a-vis the assembly is much weaker than the position of the Rajya Sabha vis-a-vis the Lok Sabha. The Rajya Sabha has equal powers with the Lok Sabha in all spheres except financial matters and with regard to the control over the Government. On the other hand, the council is subordinate to the assembly in all respects. Thus, the predominance of the assembly over the council is fully established.
-
Question 8 of 35
8. Question
Which one of the following State has a bicameral legislature?
Correct
Solution (a)
Six states have state legislative councils-
- Andhra Pradesh
- Telangana
- Uttar Pradesh
- Bihar
- Maharashtra
- Karnataka
STATE LEGISLATIVE COUNCIL:
- India has a bicameral legislature due to its federal nature where just as the parliament has two houses, states can have state legislative council in addition to state legislative assembly through article 169 of the constitution.
- Six states have state legislative councils namely Andhra Pradesh, Telangana, Uttar Pradesh, Bihar, Maharashtra and Karnataka. Recently, Jammu and Kashmir legislative council has been abolished through J&K reorganization bill 2019 which reduced the state to union territories of J&K and Ladakh.
- Article 169 provides parliament to create or abolish state legislative council by a simple majority if the legislative assembly of the concerned state passes a resolution to that effect by special majority (of two thirds of members present and voting).
- Article 171 states that state legislative council shall not have more than more than one third of the total strength of state assembly and not less than 40 members. SLC is a permanent and continuing body, with a tenure of 6 years where one third members retire every two years.
- Legislative powers of SLC are not expanse like Rajya Sabha which can mend non-financial bills while SLC suggestions can be overridden by assembly. SLC chairperson is chosen from its members itself unlike Vice president in Rajya Sabha. SLC members cannot vote in the elections for vice president and the president.
Incorrect
Solution (a)
Six states have state legislative councils-
- Andhra Pradesh
- Telangana
- Uttar Pradesh
- Bihar
- Maharashtra
- Karnataka
STATE LEGISLATIVE COUNCIL:
- India has a bicameral legislature due to its federal nature where just as the parliament has two houses, states can have state legislative council in addition to state legislative assembly through article 169 of the constitution.
- Six states have state legislative councils namely Andhra Pradesh, Telangana, Uttar Pradesh, Bihar, Maharashtra and Karnataka. Recently, Jammu and Kashmir legislative council has been abolished through J&K reorganization bill 2019 which reduced the state to union territories of J&K and Ladakh.
- Article 169 provides parliament to create or abolish state legislative council by a simple majority if the legislative assembly of the concerned state passes a resolution to that effect by special majority (of two thirds of members present and voting).
- Article 171 states that state legislative council shall not have more than more than one third of the total strength of state assembly and not less than 40 members. SLC is a permanent and continuing body, with a tenure of 6 years where one third members retire every two years.
- Legislative powers of SLC are not expanse like Rajya Sabha which can mend non-financial bills while SLC suggestions can be overridden by assembly. SLC chairperson is chosen from its members itself unlike Vice president in Rajya Sabha. SLC members cannot vote in the elections for vice president and the president.
-
Question 9 of 35
9. Question
Who among the following can be appointed by a Governor but can be removed only by the President of India?
- Chairman of State Public Service Commission.
- Chairman of Joint State Public Service Commission.
- State Election Commissioner.
Select the correct answer using the code given below:
Correct
Solution (c)
Statement 1 Statement 2 Statement 3 Correct Incorrect Correct The chairman or a member of a SPSC can be removed from the office by the President only in the manner and on the grounds mentioned in the Constitution. Therefore, they enjoy the security of tenure. Joint State Public Service Commission are appointed and removed by the president. JSPSC can be created by an act of parliament on the request of the state legislative concerned. State election commission can be removed only by the president on the basis of resolution passed by both the houses of parliament with a two-third of majority. Note:STATE PUBLIC SERVICE COMMISSION:
- A State Public Service Commission consists of appointed by the governor of the state.
- Article 315 to 323 of the constitution deals with the composition, appointment and removals of the members.
- SPSC can be removed only by the president not by the governor.
JOINT STATE PUBLIC SERVICE COMMISSION:
- The constitution provision for the establishment of a joint state public service commission for two or more states. JSPSC can be created by an act of parliament on the request of the state legislatures concerned.
- It is statutory not a constitutional body.
- Appointed by the president and removed by the president only.
- A JSPSC presents its annual performance reports to each of the concerned state governors.
Incorrect
Solution (c)
Statement 1 Statement 2 Statement 3 Correct Incorrect Correct The chairman or a member of a SPSC can be removed from the office by the President only in the manner and on the grounds mentioned in the Constitution. Therefore, they enjoy the security of tenure. Joint State Public Service Commission are appointed and removed by the president. JSPSC can be created by an act of parliament on the request of the state legislative concerned. State election commission can be removed only by the president on the basis of resolution passed by both the houses of parliament with a two-third of majority. Note:STATE PUBLIC SERVICE COMMISSION:
- A State Public Service Commission consists of appointed by the governor of the state.
- Article 315 to 323 of the constitution deals with the composition, appointment and removals of the members.
- SPSC can be removed only by the president not by the governor.
JOINT STATE PUBLIC SERVICE COMMISSION:
- The constitution provision for the establishment of a joint state public service commission for two or more states. JSPSC can be created by an act of parliament on the request of the state legislatures concerned.
- It is statutory not a constitutional body.
- Appointed by the president and removed by the president only.
- A JSPSC presents its annual performance reports to each of the concerned state governors.
-
Question 10 of 35
10. Question
Under which of the following conditions, the Governor is obligated by the constitution to reserve a bill for the consideration of the President?
Correct
Solution (d)
Article 200 states that governor may or may not reserve any state bill for consideration of the president (governor’s discretionary power) but is obligated to reserve the state bills which emphasizes to endanger the position of high court of the state.Thus the former is discretionary feature while the later is an obligatory duty on part of Governor.
Note:
PRESIDENT GOVERNOR With Regards to Ordinary Bill With regards to Ordinary Bill Every ordinary bill, after it is passed by both the houses of the parliament either singly or at a joint sitting to the president for his assent. He has three alternatives:
1. He/she may give his assent to the bill, the bill then becomes an act.
2. He/she withhold his assent the bill doesn’t become an act.
3. He/she may return for consideration.
If the bill is passes by both the house with or without amendments, president must give his assent to the bill.
President enjoys only a suspension veto.
The ordinary bill, after it is passed by state legislative assembly in case of unicameral legislature or by both the house in case of a bicameral legislature either in first or second instance, presented to the governor for his assent. He has four alternatives:
1. He/she may give his assent, the bill becomes an act.
2. He/she may withhold his assent, the bill doesn’t become an act.
3. He/she may return for reconsideration of the house. The bill is passed by the house or houses with or without amendment and presented to the governor for his assent, the governor must give his assent.
4. He/she may reserve the bill for the reconsideration of the president.
The governor enjoys only a suspensive veto.
With regards to money bill With regards to money bill Every money bill after it is passed by the parliament Either he can give his assent, or withhold but the bill cannot return for the reconsideration of the parliament.
Generally, President gives his assent to a money bill as it is introduced with the previous permission.
Every money bill after it is passed by the state legislative either he may give his assent, withhold but can reserve the bill for the reconsideration of the president. He cannot return the bill for the reconsideration of the state legislative.
Incorrect
Solution (d)
Article 200 states that governor may or may not reserve any state bill for consideration of the president (governor’s discretionary power) but is obligated to reserve the state bills which emphasizes to endanger the position of high court of the state.Thus the former is discretionary feature while the later is an obligatory duty on part of Governor.
Note:
PRESIDENT GOVERNOR With Regards to Ordinary Bill With regards to Ordinary Bill Every ordinary bill, after it is passed by both the houses of the parliament either singly or at a joint sitting to the president for his assent. He has three alternatives:
1. He/she may give his assent to the bill, the bill then becomes an act.
2. He/she withhold his assent the bill doesn’t become an act.
3. He/she may return for consideration.
If the bill is passes by both the house with or without amendments, president must give his assent to the bill.
President enjoys only a suspension veto.
The ordinary bill, after it is passed by state legislative assembly in case of unicameral legislature or by both the house in case of a bicameral legislature either in first or second instance, presented to the governor for his assent. He has four alternatives:
1. He/she may give his assent, the bill becomes an act.
2. He/she may withhold his assent, the bill doesn’t become an act.
3. He/she may return for reconsideration of the house. The bill is passed by the house or houses with or without amendment and presented to the governor for his assent, the governor must give his assent.
4. He/she may reserve the bill for the reconsideration of the president.
The governor enjoys only a suspensive veto.
With regards to money bill With regards to money bill Every money bill after it is passed by the parliament Either he can give his assent, or withhold but the bill cannot return for the reconsideration of the parliament.
Generally, President gives his assent to a money bill as it is introduced with the previous permission.
Every money bill after it is passed by the state legislative either he may give his assent, withhold but can reserve the bill for the reconsideration of the president. He cannot return the bill for the reconsideration of the state legislative.
-
Question 11 of 35
11. Question
When the same person is appointed the Governor for two or more states in India, which one of the following holds true?
Correct
Solution (c)
When a governor is in charge of two or more states, share of pension, allowances, salaries and emoluments are determined by president which base the proportion to be shared by states.
Note:
OFFICE OF THE GOVERNOR:
- The governor of the state is constituted by constitution with a dual role of being the head of state government executives (article 154) and act as constitutional representative of central government at the state level because president appoints the governor (federalism prevents governor from being an agent of central government).
- The goal of federalism to ensure competitive and cooperative federalism between the central and state governments lies with the office of the governor, provided he acts in an efficient and neutral manner.
- This surety to maintain federalism is ensured by the constitution in its Article 153 which states that there shall be governor in every state.
- However with the enactment of seventh constitutional amendment act which entitled state reorganization based separation of state based on linguistic diversity, sharing of common offices and high courts came by. This paved the way for two or more states to have a common governor.
- Transfer of the governor process includes transfer done by the president to his remaining tenure in an adjacent state. Share of pension, allowances, salaries and emoluments are determined by president which base the proportion to be shared by states, generally shared based on proportion of the tenure worked in their state.
- When a governor is in charge of two or more states, share of pension, allowances, salaries and emoluments are determined by president which base the proportion to be shared by states.
Incorrect
Solution (c)
When a governor is in charge of two or more states, share of pension, allowances, salaries and emoluments are determined by president which base the proportion to be shared by states.
Note:
OFFICE OF THE GOVERNOR:
- The governor of the state is constituted by constitution with a dual role of being the head of state government executives (article 154) and act as constitutional representative of central government at the state level because president appoints the governor (federalism prevents governor from being an agent of central government).
- The goal of federalism to ensure competitive and cooperative federalism between the central and state governments lies with the office of the governor, provided he acts in an efficient and neutral manner.
- This surety to maintain federalism is ensured by the constitution in its Article 153 which states that there shall be governor in every state.
- However with the enactment of seventh constitutional amendment act which entitled state reorganization based separation of state based on linguistic diversity, sharing of common offices and high courts came by. This paved the way for two or more states to have a common governor.
- Transfer of the governor process includes transfer done by the president to his remaining tenure in an adjacent state. Share of pension, allowances, salaries and emoluments are determined by president which base the proportion to be shared by states, generally shared based on proportion of the tenure worked in their state.
- When a governor is in charge of two or more states, share of pension, allowances, salaries and emoluments are determined by president which base the proportion to be shared by states.
-
Question 12 of 35
12. Question
Consider the following statements:
- The Governor can pardon a sentence of any person convicted of any offence against a state law.
- The Governor cannot pardon the death sentence even if a state law prescribes for it.
Which of the statements given above is/are correct?
Correct
Solution (c)
Statement 1 Statement 2 Correct Correct The Governor can grant pardons to any person convicted of any offence against (Except Death sentence) any law relating to a matter to which the executive power of the state extends. The Governor cannot pardon a death sentence. Even if a state law prescribes for death sentence, the power to grant pardon lies with the President and not the governor. But, the governor can suspend, remit or commute a death sentence. Note:
Comparing Pardoning Powers of President and Governor:
President Governor He can pardon, reprieve, respite, remit, suspend or commute the punishment or sentence of any person convicted of any offence against a Central law. He can pardon, reprieve, respite, remit, suspend or commute the punishment or sentence of convicted of any person offence against a state law. He can pardon, reprieve, respite, remit, suspend or commute a death sentence. He is the only authority to pardon a death sentence. He cannot pardon a death sentence. Even if a state law prescribes for death sentence, the power to grant pardon lies with the President and not the governor. But the governor can suspend, remit or commute a death sentence. He can grant pardon, reprieve, respite, suspension, remission or commutation in respect to punishment or sentence by a court-martial (military court). He does not possess any such power. Incorrect
Solution (c)
Statement 1 Statement 2 Correct Correct The Governor can grant pardons to any person convicted of any offence against (Except Death sentence) any law relating to a matter to which the executive power of the state extends. The Governor cannot pardon a death sentence. Even if a state law prescribes for death sentence, the power to grant pardon lies with the President and not the governor. But, the governor can suspend, remit or commute a death sentence. Note:
Comparing Pardoning Powers of President and Governor:
President Governor He can pardon, reprieve, respite, remit, suspend or commute the punishment or sentence of any person convicted of any offence against a Central law. He can pardon, reprieve, respite, remit, suspend or commute the punishment or sentence of convicted of any person offence against a state law. He can pardon, reprieve, respite, remit, suspend or commute a death sentence. He is the only authority to pardon a death sentence. He cannot pardon a death sentence. Even if a state law prescribes for death sentence, the power to grant pardon lies with the President and not the governor. But the governor can suspend, remit or commute a death sentence. He can grant pardon, reprieve, respite, suspension, remission or commutation in respect to punishment or sentence by a court-martial (military court). He does not possess any such power. -
Question 13 of 35
13. Question
Consider the following statements regarding Governor:
- The constitution of India provides that, while appointing the governor, the president is required to consult the chief minister of the state concerned.
- The office of Governor of a state is not an employment under the Central government.
Which of the statements given above is/are correct?
Correct
Solution (b)
Statement 1 Statement 2 Incorrect Correct It is a convention and not a Constitutional provision that, while appointing the governor, the president is required to consult the chief minister of the state concerned, so that the smooth functioning of the constitutional machinery in the state is ensured. As held by the Supreme Court in 1979, the office of Governor of a state is not an employment under the Central government. It is an independent constitutional office and is not under the control of or subordinate to the Central government.
Note:
APPOINTMENT OF GOVERNOR:
- He is appointed by the president by warrant under his hand and seal. In a way, he is a nominee of the Central government.
- But, as held by the Supreme Court in 1979, the office of governor of a state is not an employment under the Central government.
- It is an independent constitutional office and is not under the control of or subordinate to the Central government.
- The Constitution lays down only two qualifications for the appointment of a person as a governor. These are:
- He should be a citizen of India.
- He should have completed the age of 35 years.
- Additionally, two conventions have also developed in this regard over the years.
- He should be an outsider, that is, he should not belong to the state where he is appointed, so that he is free from the local politics.
- While appointing the governor, the president is required to consult the chief minister of the state concerned, so that the smooth functioning of the constitutional machinery in the state is ensured. However, both the conventions have been violated in some of the cases.
Incorrect
Solution (b)
Statement 1 Statement 2 Incorrect Correct It is a convention and not a Constitutional provision that, while appointing the governor, the president is required to consult the chief minister of the state concerned, so that the smooth functioning of the constitutional machinery in the state is ensured. As held by the Supreme Court in 1979, the office of Governor of a state is not an employment under the Central government. It is an independent constitutional office and is not under the control of or subordinate to the Central government.
Note:
APPOINTMENT OF GOVERNOR:
- He is appointed by the president by warrant under his hand and seal. In a way, he is a nominee of the Central government.
- But, as held by the Supreme Court in 1979, the office of governor of a state is not an employment under the Central government.
- It is an independent constitutional office and is not under the control of or subordinate to the Central government.
- The Constitution lays down only two qualifications for the appointment of a person as a governor. These are:
- He should be a citizen of India.
- He should have completed the age of 35 years.
- Additionally, two conventions have also developed in this regard over the years.
- He should be an outsider, that is, he should not belong to the state where he is appointed, so that he is free from the local politics.
- While appointing the governor, the president is required to consult the chief minister of the state concerned, so that the smooth functioning of the constitutional machinery in the state is ensured. However, both the conventions have been violated in some of the cases.
-
Question 14 of 35
14. Question
Which one of the following suggested that the Governor should be an eminent person from outside the State and should be a detached figure without intense political links or should not have taken part in politics in the recent past?
Correct
Solution (c)
Sarkaria Commission (1983):
- The Commission warned that the Emergency provision under Article 356 should be used ‘very sparingly, in extreme cases’ as the last resort when all available alternatives failed.
- Moreover, the Legislative Assembly should not be dissolved until the Emergency proclamation under Article 356 has been laid before and considered by Parliament.
- To prevent prejudices in the functioning of the State governments, it suggested that the Governor should be an eminent person from outside the State and should be a detached figure without intense political links or should not have taken part in politics in the recent past.
Incorrect
Solution (c)
Sarkaria Commission (1983):
- The Commission warned that the Emergency provision under Article 356 should be used ‘very sparingly, in extreme cases’ as the last resort when all available alternatives failed.
- Moreover, the Legislative Assembly should not be dissolved until the Emergency proclamation under Article 356 has been laid before and considered by Parliament.
- To prevent prejudices in the functioning of the State governments, it suggested that the Governor should be an eminent person from outside the State and should be a detached figure without intense political links or should not have taken part in politics in the recent past.
-
Question 15 of 35
15. Question
Consider the following statements with respect to the legislative powers of the Governor of a state:
- The Governor can summon and prorogue the State Assembly.
- The Governor can adjourn the sittings of the State Assembly.
- The Governor addresses the first session of the Legislative Assembly after elections.
- The Governor causes to lay the annual budget in the State Assembly.
How many of the above statements are correct?
Correct
Solution (c)
Statement 1 Statement 2 Statement 3 Statement 4 Correct Incorrect Correct Correct The Governor is to summon the House or each House of the State Legislature, if it is a bicameral legislature, to meet at such time and place as he deems fit. Six months should not, however, elapse between its last sitting in one session and the first in the next session. He may prorogue the Houses or either House and dissolve the Legislative Assembly. Speaker adjourns the sitting of the state assembly. He can address either or both of the Houses, assembled together at the commencement of the first session after each General Election and also, at the commencement of the first session each year. He sees that the Annual Financial Statement (state budget) is laid before the state legislature. Note:
- The Bills passed by the State legislature require his assent. He can withhold his assent and return the Bill (other than a Money Bill) to the State legislature for reconsideration. But if the House passes the Bill second time, with or without amendment, he must accord his assent to it.
- He is empowered to reserve certain Bills for the assent of the President. For instance, the Bills providing for compulsory acquisition of the property or curtailing the powers of the High Court have to be so reserved for President’s consent.
- He nominates persons, having special knowledge or practical experience in respect of such matters, as Literature, Art, Science, Co-operative Movement and Social Service.
- He nominates some members of the Anglo-Indian Community if he finds the latter inadequately represented.
- On the advice of the Election commission, he is authorized to decide questions arising about the disqualification of any member of either House.
- He can issue ordinances during the recess of the legislature if some eventuality arises. These ordinances cease to operate at the expiration of six weeks from the reassembly of the Legislature, or earlier if a resolution disapproving such an ordinance is passed by the Legislative Assembly and agreed to by the Legislative Council. Governor’s ordinances are subject to certain restrictions. If they relate to any matter in respect of which a Bill would have required the President’s previous sanction or his assent after reservation, the Governor has no power to issue them except on the President’s instruction.
- He may also send messages to the House or Houses on a Bill pending in the legislature or otherwise.
- He can get the State Assembly suspended while recommending to the President the taking over of the State Administration. Such a step is taken with the hope to reinstall popular ministry at an early date. This was done by the Governor of Uttar Pradesh on June 13, 1973 and Governor of Punjab on October 4, 1983.
- Likewise he can recommend for keeping the State Legislature in suspended animation, while installation of the President’s rule.
Incorrect
Solution (c)
Statement 1 Statement 2 Statement 3 Statement 4 Correct Incorrect Correct Correct The Governor is to summon the House or each House of the State Legislature, if it is a bicameral legislature, to meet at such time and place as he deems fit. Six months should not, however, elapse between its last sitting in one session and the first in the next session. He may prorogue the Houses or either House and dissolve the Legislative Assembly. Speaker adjourns the sitting of the state assembly. He can address either or both of the Houses, assembled together at the commencement of the first session after each General Election and also, at the commencement of the first session each year. He sees that the Annual Financial Statement (state budget) is laid before the state legislature. Note:
- The Bills passed by the State legislature require his assent. He can withhold his assent and return the Bill (other than a Money Bill) to the State legislature for reconsideration. But if the House passes the Bill second time, with or without amendment, he must accord his assent to it.
- He is empowered to reserve certain Bills for the assent of the President. For instance, the Bills providing for compulsory acquisition of the property or curtailing the powers of the High Court have to be so reserved for President’s consent.
- He nominates persons, having special knowledge or practical experience in respect of such matters, as Literature, Art, Science, Co-operative Movement and Social Service.
- He nominates some members of the Anglo-Indian Community if he finds the latter inadequately represented.
- On the advice of the Election commission, he is authorized to decide questions arising about the disqualification of any member of either House.
- He can issue ordinances during the recess of the legislature if some eventuality arises. These ordinances cease to operate at the expiration of six weeks from the reassembly of the Legislature, or earlier if a resolution disapproving such an ordinance is passed by the Legislative Assembly and agreed to by the Legislative Council. Governor’s ordinances are subject to certain restrictions. If they relate to any matter in respect of which a Bill would have required the President’s previous sanction or his assent after reservation, the Governor has no power to issue them except on the President’s instruction.
- He may also send messages to the House or Houses on a Bill pending in the legislature or otherwise.
- He can get the State Assembly suspended while recommending to the President the taking over of the State Administration. Such a step is taken with the hope to reinstall popular ministry at an early date. This was done by the Governor of Uttar Pradesh on June 13, 1973 and Governor of Punjab on October 4, 1983.
- Likewise he can recommend for keeping the State Legislature in suspended animation, while installation of the President’s rule.
-
Question 16 of 35
16. Question
Which of the following was not a reason behind adopting the present system of appointment of Governor as against the system of direct election?
Correct
Solution (a)
Appointment of the Governor by President without any direct elections was chosen because of the following reasons:
- The direct election of the governor is incompatible with the parliamentary system established in the states.
- The mode of direct election is more likely to create conflicts between the governor and the chief minister.
- The governor being only a constitutional (nominal) head, there is no point in making elaborate arrangements for his election and spending huge amount of money.
- The election of a governor would be entirely on personal issues. Hence, it is not in the national interest to involve a large number of voters in such an election.
- An elected governor would naturally belong to a party and would not be a neutral person and an impartial head.
- The election of the governor would create separatist tendencies and thus affect the political stability and unity of the country.
- The system of presidential nomination enables the Centre to maintain its control over the states.
- The direct election of the governor creates a serious problem of leadership at the time of a general election in the state.
- The chief minister would like his nominee to contest for governorship. Hence, a second-rate man of the ruling party is elected as governor.
Incorrect
Solution (a)
Appointment of the Governor by President without any direct elections was chosen because of the following reasons:
- The direct election of the governor is incompatible with the parliamentary system established in the states.
- The mode of direct election is more likely to create conflicts between the governor and the chief minister.
- The governor being only a constitutional (nominal) head, there is no point in making elaborate arrangements for his election and spending huge amount of money.
- The election of a governor would be entirely on personal issues. Hence, it is not in the national interest to involve a large number of voters in such an election.
- An elected governor would naturally belong to a party and would not be a neutral person and an impartial head.
- The election of the governor would create separatist tendencies and thus affect the political stability and unity of the country.
- The system of presidential nomination enables the Centre to maintain its control over the states.
- The direct election of the governor creates a serious problem of leadership at the time of a general election in the state.
- The chief minister would like his nominee to contest for governorship. Hence, a second-rate man of the ruling party is elected as governor.
-
Question 17 of 35
17. Question
Consider the following statements about the office of Governor:
- A Member of the Parliament cannot be appointed as Governor of a state.
- The Governor shall make rules for the more convenient transaction of the business of the government of the state.
- The Governor appoints the Advocate General of a state.
How many of the above statements are correct?
Correct
Solution (b)
Statement 1 Statement 2 Statement 3 Incorrect Correct Correct He should not be a member of either House of Parliament or a House of the state legislature. If any such person is appointed as governor, he is deemed to have vacated his seat in that House on the date on which he enters upon his office as the governor. Thus a member of Parliament can be appointed as governor.However as soon as he enters the office of governor his seat of MP is declared vacant by default.
He can make rules for the more convenient transaction of the business of a state government and for the allocation among the ministers of the said business. He appoints the advocate general of a state and determines his remuneration. The advocate general holds office during the pleasure of the governor. Note:
- Article 153 provides for Governor for each State.
- The Constitution, under article 158, lays down the following conditions for the governor’s office:
- He should not be a member of either House of Parliament or a House of the state legislature. If any such person is appointed as governor, he is deemed to have vacated his seat in that House on the date on which he enters upon his office as the governor.
- He should not hold any other office of profit.
- He is entitled without payment of rent to the use of his official residence (the Raj Bhavan).
Executive powers and functions of the Governor:
- All executive actions of the government of a state are formally taken in his name.
- He can make rules specifying the manner in which the Orders and other instruments made and executed in his name shall be authenticated.
- He can make rules for the more convenient transaction of the business of a state government and for the allocation among the ministers of the said business.
- He appoints the chief minister and other ministers. They also hold office during his pleasure.
- There should be a Tribal Welfare minister in the states of Chhattisgarh, Jharkhand, Madhya Pradesh and Odisha appointed by him. The state of Bihar was excluded from this provision by the 94th Amendment Act of 2006.
- He appoints the advocate general of a state and determines his remuneration. The advocate general holds office during the pleasure of the governor.
Attorney General is appointed by the President of India.
Incorrect
Solution (b)
Statement 1 Statement 2 Statement 3 Incorrect Correct Correct He should not be a member of either House of Parliament or a House of the state legislature. If any such person is appointed as governor, he is deemed to have vacated his seat in that House on the date on which he enters upon his office as the governor. Thus a member of Parliament can be appointed as governor.However as soon as he enters the office of governor his seat of MP is declared vacant by default.
He can make rules for the more convenient transaction of the business of a state government and for the allocation among the ministers of the said business. He appoints the advocate general of a state and determines his remuneration. The advocate general holds office during the pleasure of the governor. Note:
- Article 153 provides for Governor for each State.
- The Constitution, under article 158, lays down the following conditions for the governor’s office:
- He should not be a member of either House of Parliament or a House of the state legislature. If any such person is appointed as governor, he is deemed to have vacated his seat in that House on the date on which he enters upon his office as the governor.
- He should not hold any other office of profit.
- He is entitled without payment of rent to the use of his official residence (the Raj Bhavan).
Executive powers and functions of the Governor:
- All executive actions of the government of a state are formally taken in his name.
- He can make rules specifying the manner in which the Orders and other instruments made and executed in his name shall be authenticated.
- He can make rules for the more convenient transaction of the business of a state government and for the allocation among the ministers of the said business.
- He appoints the chief minister and other ministers. They also hold office during his pleasure.
- There should be a Tribal Welfare minister in the states of Chhattisgarh, Jharkhand, Madhya Pradesh and Odisha appointed by him. The state of Bihar was excluded from this provision by the 94th Amendment Act of 2006.
- He appoints the advocate general of a state and determines his remuneration. The advocate general holds office during the pleasure of the governor.
Attorney General is appointed by the President of India.
-
Question 18 of 35
18. Question
Consider the following statements regarding the Chief Minister (CM):
- He can recommend the dissolution of the legislative assembly to the governor at any time.
- He acts as chairman of the concerned zonal council by rotation.
- He can ask a minister to resign or advise the governor to dismiss him in case of a difference of opinion.
- He is a member of the Governing Council of NITI Aayog.
How many of the above statements are correct?
Correct
Solution (c)
Statement 1 Statement 2 Statement 3 Statement 4 Correct Incorrect Correct Correct He can recommend the dissolution of the legislative assembly to the governor at any time. He acts as a vice-chairman of the concerned zonal council by rotation, holding office for a period of one year at a time. He can ask a minister to resign or advise the governor to dismiss him in case of difference of opinion. He is a member of the Inter-State Council and the Governing Council of NITI Aayog, both headed by the Prime Minister. Note:
- In the scheme of the parliamentary system of government, provided by the Constitution, the governor is the nominal executive authority and the chief minister is the real executive authority.
- Article 163 provides for the office of Chief Minister as the head of the council of ministers to aid and advise the Governor.
- The Chief Minister also performs the following functions:
- He is the chairman of the State Planning Board.
- He acts as a vice-chairman of the concerned zonal council by rotation, holding office for a period of one year at a time.
- He is a member of the Inter-State Council and the Governing Council of NITI Aayog, both headed by the prime minister.
The Chief Minister enjoys the following powers as the leader of the house:
- He advises the governor with regard to the summoning and proroguing of the sessions of the state legislature.
- He can recommend the dissolution of the legislative assembly to the governor at any time.
- He announces the government policies on the floor of the house.
The Chief Minister enjoys the following powers as head of the state council of ministers:
- The governor appoints only those persons as ministers who are recommended by the Chief Minister.
- He allocates and reshuffles the portfolios among ministers.
- He can ask a minister to resign or advise the governor to dismiss him in case of difference of opinion.
- He presides over the meetings of the council of ministers and influences its decisions.
Incorrect
Solution (c)
Statement 1 Statement 2 Statement 3 Statement 4 Correct Incorrect Correct Correct He can recommend the dissolution of the legislative assembly to the governor at any time. He acts as a vice-chairman of the concerned zonal council by rotation, holding office for a period of one year at a time. He can ask a minister to resign or advise the governor to dismiss him in case of difference of opinion. He is a member of the Inter-State Council and the Governing Council of NITI Aayog, both headed by the Prime Minister. Note:
- In the scheme of the parliamentary system of government, provided by the Constitution, the governor is the nominal executive authority and the chief minister is the real executive authority.
- Article 163 provides for the office of Chief Minister as the head of the council of ministers to aid and advise the Governor.
- The Chief Minister also performs the following functions:
- He is the chairman of the State Planning Board.
- He acts as a vice-chairman of the concerned zonal council by rotation, holding office for a period of one year at a time.
- He is a member of the Inter-State Council and the Governing Council of NITI Aayog, both headed by the prime minister.
The Chief Minister enjoys the following powers as the leader of the house:
- He advises the governor with regard to the summoning and proroguing of the sessions of the state legislature.
- He can recommend the dissolution of the legislative assembly to the governor at any time.
- He announces the government policies on the floor of the house.
The Chief Minister enjoys the following powers as head of the state council of ministers:
- The governor appoints only those persons as ministers who are recommended by the Chief Minister.
- He allocates and reshuffles the portfolios among ministers.
- He can ask a minister to resign or advise the governor to dismiss him in case of difference of opinion.
- He presides over the meetings of the council of ministers and influences its decisions.
-
Question 19 of 35
19. Question
Which of the following statements is/ are correct with reference to the executive powers of the Governor?
- He appoints the State Election Commissioner and determines his conditions of service.
- He appoints and removes the Chairman and Members of the State Public Service Commission.
Select the correct answer using the code given below:
Correct
Solution (a)
Statement 1 Statement 2 Correct Incorrect He appoints the State Election Commissioner and determines his service of condition and tenure. However, State Election Commissioner can be removed only in like manner and on the grounds as a judge of a high court. He appoints the Chairman and member of the state public service Commission. However, they can be removed only by the President and not by the Governor. Note:
Executive powers and function of the Governor:
- All the executive actions that the state government take, are to be taken in his name.
- He can make rules specifying the manners in which orders and other instruments made and executed in his name shall be authenticated.
- He can make rules for more convenient transactions of the business of a state government and for the allocation among the ministers of the said business.
- Chief Ministers and other ministers of the states are appointed by him. They hold office during the pleasure of the Governor
- They should also appoint Tribal Welfare Minister in the states of Chhattisgarh, Jharkhand, Madhya Pradesh, Odisha.
- He appoints the advocate general of states and determines their remuneration.
- He appoints the State Election Commissioner and determines his service of condition and tenure. However, State Election Commissioner can be removed only in like manner and on the grounds as a judge of a high court.
- He appoints the Chairman and member of the state public service Commission. However, they can be removed only by the President and not by the Governor.
- He can seek any information relating to the administration of affairs of the state and proposal for legislation from the Chief Minister.
- He can require the Chief Minister to submit for the consideration of the council of ministers any matter on which a decision had been taken by a minister but which has not been considered by the council.
- A constitutional emergency in the state is recommended to the President by him.
- The governor enjoys extensive executive powers as an agent of the President during the President’s rule in the state.
Incorrect
Solution (a)
Statement 1 Statement 2 Correct Incorrect He appoints the State Election Commissioner and determines his service of condition and tenure. However, State Election Commissioner can be removed only in like manner and on the grounds as a judge of a high court. He appoints the Chairman and member of the state public service Commission. However, they can be removed only by the President and not by the Governor. Note:
Executive powers and function of the Governor:
- All the executive actions that the state government take, are to be taken in his name.
- He can make rules specifying the manners in which orders and other instruments made and executed in his name shall be authenticated.
- He can make rules for more convenient transactions of the business of a state government and for the allocation among the ministers of the said business.
- Chief Ministers and other ministers of the states are appointed by him. They hold office during the pleasure of the Governor
- They should also appoint Tribal Welfare Minister in the states of Chhattisgarh, Jharkhand, Madhya Pradesh, Odisha.
- He appoints the advocate general of states and determines their remuneration.
- He appoints the State Election Commissioner and determines his service of condition and tenure. However, State Election Commissioner can be removed only in like manner and on the grounds as a judge of a high court.
- He appoints the Chairman and member of the state public service Commission. However, they can be removed only by the President and not by the Governor.
- He can seek any information relating to the administration of affairs of the state and proposal for legislation from the Chief Minister.
- He can require the Chief Minister to submit for the consideration of the council of ministers any matter on which a decision had been taken by a minister but which has not been considered by the council.
- A constitutional emergency in the state is recommended to the President by him.
- The governor enjoys extensive executive powers as an agent of the President during the President’s rule in the state.
-
Question 20 of 35
20. Question
With reference to the First Past the Post (FPTP) system, which of the following is/are correct?
- Under this system, a winning candidate has to get more than 50% of the total votes in a constituency.
- This method is also called the Plurality System.
Select the answer:
Correct
Solution (b)
Statement 1 Statement 2 Incorrect Correct In First Past the Post (FPTP) system, whoever has more votes than all other candidates, is declared elected. The winning candidate need not secure a majority of the votes. This method is called the First Past the Post (FPTP) system. In the electoral race, the candidate who is ahead of others, who crosses the winning post first of all, is the winner. This method is also called the Plurality System. This is the method of election prescribed by the Constitution. Under this system, The votes that go to all the losing candidates go ‘waste’, for those candidates or parties get no seat from those votes.
Incorrect
Solution (b)
Statement 1 Statement 2 Incorrect Correct In First Past the Post (FPTP) system, whoever has more votes than all other candidates, is declared elected. The winning candidate need not secure a majority of the votes. This method is called the First Past the Post (FPTP) system. In the electoral race, the candidate who is ahead of others, who crosses the winning post first of all, is the winner. This method is also called the Plurality System. This is the method of election prescribed by the Constitution. Under this system, The votes that go to all the losing candidates go ‘waste’, for those candidates or parties get no seat from those votes.
-
Question 21 of 35
21. Question
Consider the following statements about the Atal Innovation Mission:
- Its objective is to create and promote an ecosystem of innovation and entrepreneurship across the country.
- It is implemented by the Ministry of Skill Development and Entrepreneurship.
- Its components include Atal Tinkering Labs, Atal Incubation Centres, and Atal New India Challenge.
How many of the above statements are correct?
Correct
Solution (b)
- The Atal Innovation Mission is the Government of India’s flagship initiative to promote a culture of innovation and entrepreneurship in the country and was set up in 2016.
- Its objective is to create and promote an ecosystem of innovation and entrepreneurship across the country at school, university, research institutions, MSME, and industry levels. Hence statement 1 is correct.
- All the initiatives of AIM are currently monitored and managed systematically using real-time MIS systems and dynamic dashboards.
- It is implemented by the NITI Aayog. Hence statement 2 is incorrect.
- It has multiple programs to encourage and support innovation in the country.
- Its components include Atal Tinkering Labs, Atal Incubation Centres, Mentor of Change Program, Atal Community Innovation Center Atal Research & Innovation for Small Enterprises (ARISE), and Atal New India Challenge. Hence statement 3 is correct.
Incorrect
Solution (b)
- The Atal Innovation Mission is the Government of India’s flagship initiative to promote a culture of innovation and entrepreneurship in the country and was set up in 2016.
- Its objective is to create and promote an ecosystem of innovation and entrepreneurship across the country at school, university, research institutions, MSME, and industry levels. Hence statement 1 is correct.
- All the initiatives of AIM are currently monitored and managed systematically using real-time MIS systems and dynamic dashboards.
- It is implemented by the NITI Aayog. Hence statement 2 is incorrect.
- It has multiple programs to encourage and support innovation in the country.
- Its components include Atal Tinkering Labs, Atal Incubation Centres, Mentor of Change Program, Atal Community Innovation Center Atal Research & Innovation for Small Enterprises (ARISE), and Atal New India Challenge. Hence statement 3 is correct.
-
Question 22 of 35
22. Question
Consider the following statements about the Scheme for Support to Poor Prisoners:
- It aims to provide financial support to poor prisoners who are not being released from jails as they are unable to pay the fines imposed on them.
- It was launched by the Ministry of Home Affairs (MHA).
- An Empowered Committee is to be constituted in each district to identify under-trials who need financial support.
- The National Crime Records Bureau (NCRB) has been designated as the Central Nodal Agency (CNA) for this scheme.
How many of the above statements are correct?
Correct
Solution (d)
- The Scheme for Support to Poor Prisoners aims to provide financial support to poor prisoners who are not being released from jails as they are unable to pay the fines imposed on them or are not able to secure bail due to financial constraints. Hence statement 1 is correct.
- It was launched by the Ministry of Home Affairs (MHA). Hence statement 2 is correct.
- According to the National Crime Records Bureau (NCRB) data of 2020, roughly 76% of individuals in Indian prisons were under-trials. Among these, approximately 68% were either illiterate or school dropouts.
- An Empowered Committee is to be constituted in each district to identify under-trials who need financial support. Hence statement 3 is correct.
- Funds to the States/UTs will be provided through the Central Nodal Agency (CNA).
- The National Crime Records Bureau (NCRB) has been designated as the Central Nodal Agency (CNA) for this scheme. Hence statement 4 is correct.
Incorrect
Solution (d)
- The Scheme for Support to Poor Prisoners aims to provide financial support to poor prisoners who are not being released from jails as they are unable to pay the fines imposed on them or are not able to secure bail due to financial constraints. Hence statement 1 is correct.
- It was launched by the Ministry of Home Affairs (MHA). Hence statement 2 is correct.
- According to the National Crime Records Bureau (NCRB) data of 2020, roughly 76% of individuals in Indian prisons were under-trials. Among these, approximately 68% were either illiterate or school dropouts.
- An Empowered Committee is to be constituted in each district to identify under-trials who need financial support. Hence statement 3 is correct.
- Funds to the States/UTs will be provided through the Central Nodal Agency (CNA).
- The National Crime Records Bureau (NCRB) has been designated as the Central Nodal Agency (CNA) for this scheme. Hence statement 4 is correct.
-
Question 23 of 35
23. Question
Consider the following statements about ‘White Hydrogen’:
- It is a naturally occurring form of hydrogen found in the Earth’s crust.
- It is a clean energy source as it produces only water when it is burned.
Which of the statements given above is/are correct?
Correct
Solution (c)
In northeastern France, scientists searching for fossil fuels beneath the Earth’s surface stumbled upon a vast reservoir of hydrogen, specifically “white hydrogen”. This remarkable discovery, estimated to range from 6 to 250 million metric tons, stands as one of the largest deposits of white hydrogen ever found.
- White Hydrogen is a naturally occurring form of hydrogen found in the Earth’s crust. Hence statement 1 is correct.
- It is also known as “natural,” “gold,” or “geologic” hydrogen.
- It has been named ‘white’ since the “production doesn’t generate greenhouse gases”.
- It is cost-efficient as white hydrogen’s estimated cost is around $1 per kilogram, significantly lower than green hydrogen, which costs about $6 per kilogram, making it a more affordable clean energy source.
- It is a clean energy source as it produces only water when it is burned. Hence statement 2 is correct.
- It causes no CO2 emissions when used as a fuel.
Incorrect
Solution (c)
In northeastern France, scientists searching for fossil fuels beneath the Earth’s surface stumbled upon a vast reservoir of hydrogen, specifically “white hydrogen”. This remarkable discovery, estimated to range from 6 to 250 million metric tons, stands as one of the largest deposits of white hydrogen ever found.
- White Hydrogen is a naturally occurring form of hydrogen found in the Earth’s crust. Hence statement 1 is correct.
- It is also known as “natural,” “gold,” or “geologic” hydrogen.
- It has been named ‘white’ since the “production doesn’t generate greenhouse gases”.
- It is cost-efficient as white hydrogen’s estimated cost is around $1 per kilogram, significantly lower than green hydrogen, which costs about $6 per kilogram, making it a more affordable clean energy source.
- It is a clean energy source as it produces only water when it is burned. Hence statement 2 is correct.
- It causes no CO2 emissions when used as a fuel.
-
Question 24 of 35
24. Question
Consider the following statements about ‘Pichwai Paintings’:
- They are a traditional form of Indian art that originated in the state of Maharashtra.
- They predominantly depict stories and legends of Lord Krishna.
- They use natural colours made from minerals and plant extracts.
How many of the above statements are correct?
Correct
Solution (b)
- Pichwai Paintings are a traditional form of Indian art that originated in the state of Rajasthan. Hence statement 1 is incorrect.
- They are usually done on khadi.
- They predominantly depict stories and legends of Lord Krishna. Hence statement 2 is correct.
- They are hung behind the idol of Shrinathji, a local form of Krishna and the centre of Pushtimarg worship.
- They use natural colours made from minerals and plant extracts. Hence statement 3 is correct.
- Stone pigments are used for gold and silver tones. Vegetable dyes are used for brighter orange, red, chrome yellow, and kesari colours.
Incorrect
Solution (b)
- Pichwai Paintings are a traditional form of Indian art that originated in the state of Rajasthan. Hence statement 1 is incorrect.
- They are usually done on khadi.
- They predominantly depict stories and legends of Lord Krishna. Hence statement 2 is correct.
- They are hung behind the idol of Shrinathji, a local form of Krishna and the centre of Pushtimarg worship.
- They use natural colours made from minerals and plant extracts. Hence statement 3 is correct.
- Stone pigments are used for gold and silver tones. Vegetable dyes are used for brighter orange, red, chrome yellow, and kesari colours.
-
Question 25 of 35
25. Question
Consider the following statements about White Phosphorous:
- It is also known as yellow phosphorous.
- It is a waxy crystalline solid.
- White phosphorus is used for military purposes.
How many of the above statements are correct?
Correct
Solution (c)
Key facts about White Phosphorous (P4)- White phosphorus, yellow phosphorus or simply tetraphosphorus (P4) exists as molecules of phosphorus made up of four atoms in a tetrahedral structure.
- White phosphorus is a chemical waxy solid substance typically appearing yellowish or colourless, and some have described its odour as resembling garlic.
- It ignites instantly upon contact with oxygen. It is often used by militaries to illuminate battlefields, generate a smokescreen and as an incendiary.
- Once ignited, white phosphorus is very difficult to extinguish. It sticks to surfaces like skin and clothing.
- White phosphorus is harmful by all routes of exposure.
- White phosphorus can cause deep and severe burns, penetrating even through bone, and has been known to reignite after initial treatment. After exposure, the priority is to stop the burning process.
- The smoke from burning phosphorus is also harmful to the eyes and respiratory tract due to the presence of phosphoric acids and phosphine.
Incorrect
Solution (c)
Key facts about White Phosphorous (P4)- White phosphorus, yellow phosphorus or simply tetraphosphorus (P4) exists as molecules of phosphorus made up of four atoms in a tetrahedral structure.
- White phosphorus is a chemical waxy solid substance typically appearing yellowish or colourless, and some have described its odour as resembling garlic.
- It ignites instantly upon contact with oxygen. It is often used by militaries to illuminate battlefields, generate a smokescreen and as an incendiary.
- Once ignited, white phosphorus is very difficult to extinguish. It sticks to surfaces like skin and clothing.
- White phosphorus is harmful by all routes of exposure.
- White phosphorus can cause deep and severe burns, penetrating even through bone, and has been known to reignite after initial treatment. After exposure, the priority is to stop the burning process.
- The smoke from burning phosphorus is also harmful to the eyes and respiratory tract due to the presence of phosphoric acids and phosphine.
-
Question 26 of 35
26. Question
Consider the following statements about Mercury:
- It is naturally found in volcanoes, geothermal springs, geologic deposits, and the ocean.
- Exposure to it can cause severe health problems, particularly neurological and developmental issues in foetuses, infants, and children.
- It is commonly used in small-scale gold mining to separate gold from other minerals due to its ability to bind to gold.
How many of the above statements are correct?
Correct
Solution (c)
Context: According to a study, tropical birds are being affected by mercury pollution due to expanding artisanal and small-scale gold mining operations in jungle areas.
- Artisanal mining refers to small-scale, subsistence-based mining activities typically carried out by individuals or small groups using basic tools and limited machinery. This form of mining often occurs in rural or remote areas, focusing on extracting minerals or precious metals like gold, diamonds, or gemstones.
- Mercury is naturally found in volcanoes, geothermal springs, geologic deposits, and the ocean. Hence statement 1 is correct.
- Its human-related sources include coal combustion, waste incineration, industrial uses, and mining.
- Exposure to it can cause severe health problems, particularly neurological and developmental issues in foetuses, infants, and children. Hence statement 2 is correct.
- Long-term exposure can lead to damage to the brain, nervous system, and kidneys in humans.
- It is commonly used in small-scale gold mining to separate gold from other minerals due to its ability to bind to gold. Hence statement 3 is correct.
- Minamata Convention obliges government parties to take a range of actions to address mercury emissions into the air and to phase out certain mercury-containing products.
Incorrect
Solution (c)
Context: According to a study, tropical birds are being affected by mercury pollution due to expanding artisanal and small-scale gold mining operations in jungle areas.
- Artisanal mining refers to small-scale, subsistence-based mining activities typically carried out by individuals or small groups using basic tools and limited machinery. This form of mining often occurs in rural or remote areas, focusing on extracting minerals or precious metals like gold, diamonds, or gemstones.
- Mercury is naturally found in volcanoes, geothermal springs, geologic deposits, and the ocean. Hence statement 1 is correct.
- Its human-related sources include coal combustion, waste incineration, industrial uses, and mining.
- Exposure to it can cause severe health problems, particularly neurological and developmental issues in foetuses, infants, and children. Hence statement 2 is correct.
- Long-term exposure can lead to damage to the brain, nervous system, and kidneys in humans.
- It is commonly used in small-scale gold mining to separate gold from other minerals due to its ability to bind to gold. Hence statement 3 is correct.
- Minamata Convention obliges government parties to take a range of actions to address mercury emissions into the air and to phase out certain mercury-containing products.
-
Question 27 of 35
27. Question
Consider the following statements about Small Modular Reactors (SMR):
- They can be installed in locations unsuitable for larger nuclear power plants.
- They are built only on-site and not in a factory like conventional reactors.
Choose the correct code:
Correct
Solution (a)
- Small Modular Reactors (SMRs) are a class of nuclear reactors that are smaller in size compared to traditional, large-scale nuclear power plants. They are designed to be more flexible, scalable, and easily deployable in various settings.
- SMRs are smaller in size. This enables their installation in locations unsuitable for larger nuclear power plants. Hence statement 1 is correct.
- They have a power capacity of up to 300 MW per unit which is about a third of the capacity of a traditional nuclear reactor.
- Nuclear energy produced by SMRs can contribute to a reduction in greenhouse gas emissions, thereby supporting efforts to combat climate change.
- SMRs are built in a factory and not on-site like conventional reactors. Hence statement 2 is incorrect.
Incorrect
Solution (a)
- Small Modular Reactors (SMRs) are a class of nuclear reactors that are smaller in size compared to traditional, large-scale nuclear power plants. They are designed to be more flexible, scalable, and easily deployable in various settings.
- SMRs are smaller in size. This enables their installation in locations unsuitable for larger nuclear power plants. Hence statement 1 is correct.
- They have a power capacity of up to 300 MW per unit which is about a third of the capacity of a traditional nuclear reactor.
- Nuclear energy produced by SMRs can contribute to a reduction in greenhouse gas emissions, thereby supporting efforts to combat climate change.
- SMRs are built in a factory and not on-site like conventional reactors. Hence statement 2 is incorrect.
-
Question 28 of 35
28. Question
Consider the following statements:
- A consulate is a diplomatic mission that represents the government of one country in another.
- An embassy is a smaller office that is typically located in a major city of the host country.
- The ambassador is the head of the embassy and is the primary representative of the government in the host country.
- The consulate’s main purpose is to provide consular services to citizens of the home country who are travelling or living abroad.
How many of the above statements are correct?
Correct
Solution (b)
- An embassy is a diplomatic mission that represents the government of one country in another. Hence statement 1 is incorrect.
- It is usually located in the capital city of the host country.
- A consulate is a smaller office that is typically located in a major city of the host country. Hence statement 2 is incorrect.
- The ambassador is the head of the embassy and is the primary representative of the government in the host country. Hence statement 3 is correct.
- The consulate’s main purpose is to provide consular services to citizens of the home country who are travelling or living abroad. Hence statement 4 is correct.
Incorrect
Solution (b)
- An embassy is a diplomatic mission that represents the government of one country in another. Hence statement 1 is incorrect.
- It is usually located in the capital city of the host country.
- A consulate is a smaller office that is typically located in a major city of the host country. Hence statement 2 is incorrect.
- The ambassador is the head of the embassy and is the primary representative of the government in the host country. Hence statement 3 is correct.
- The consulate’s main purpose is to provide consular services to citizens of the home country who are travelling or living abroad. Hence statement 4 is correct.
-
Question 29 of 35
29. Question
Consider the following statements about ‘Enabling Communications on Real-time Environment’ (ENCORE):
- It aims to provide a seamless facility for Returning Officers to process candidate nominations, affidavits, Voter turnout, counting, results, and data management.
- It was launched by the Ministry of Home Affairs.
Choose the correct code:
Correct
Solution (a)
- Enabling Communications on Real-time Environment (ENCORE) aims to provide a seamless facility for Returning Officers to process candidate nominations, affidavits, Voter turnout, counting, results, and data management. Hence statement 1 is correct.
It was launched by the Election Commission of India. Hence statement 2 is incorrect.
Incorrect
Solution (a)
- Enabling Communications on Real-time Environment (ENCORE) aims to provide a seamless facility for Returning Officers to process candidate nominations, affidavits, Voter turnout, counting, results, and data management. Hence statement 1 is correct.
It was launched by the Election Commission of India. Hence statement 2 is incorrect.
-
Question 30 of 35
30. Question
The State Food Safety Index is an annual assessment released by the?
Correct
Solution (d)
The State Food Safety Index is an annual assessment released by the Food Safety and Standards Authority of India (FSSAI). It is a dynamic quantitative and qualitative benchmarking model that provides an objective framework for evaluating food safety across all States/UTs. It is meant to create a competitive and positive change in the food safety ecosystem in the country. The step was also taken to galvanize states and union territories to work towards ensuring safe food for citizens. Hence option d is correct.
Incorrect
Solution (d)
The State Food Safety Index is an annual assessment released by the Food Safety and Standards Authority of India (FSSAI). It is a dynamic quantitative and qualitative benchmarking model that provides an objective framework for evaluating food safety across all States/UTs. It is meant to create a competitive and positive change in the food safety ecosystem in the country. The step was also taken to galvanize states and union territories to work towards ensuring safe food for citizens. Hence option d is correct.
-
Question 31 of 35
31. Question
If the number 481*673 is completely divisible by 9, then the smallest whole number in the place of * will be?
Correct
Solution (d)
If the sum of all the digits of the given number is divisible by 9, then we decide that the given number is also divisible by 9 = (4 + 8 + 1 + x + 6 + 7 + 3) / 9 = (29 + x) / 9
If x = 3, then = (29 + 3) / 9 = 32/9 ,not divisible by 9
If x = 4, then = (29 + 4) / 9 = 33/9 , not divisible by 9
If x = 7, then = (29 + 7) / 9 = 36/9 divisible by 9
Hence the smallest whole number for x should be 7.
Incorrect
Solution (d)
If the sum of all the digits of the given number is divisible by 9, then we decide that the given number is also divisible by 9 = (4 + 8 + 1 + x + 6 + 7 + 3) / 9 = (29 + x) / 9
If x = 3, then = (29 + 3) / 9 = 32/9 ,not divisible by 9
If x = 4, then = (29 + 4) / 9 = 33/9 , not divisible by 9
If x = 7, then = (29 + 7) / 9 = 36/9 divisible by 9
Hence the smallest whole number for x should be 7.
-
Question 32 of 35
32. Question
x and y are real numbers satisfying the conditions 3 < x < 4 and – 9 < y < – 8. Which of the following expressions will have the least value?
Correct
Solution (c)
From the given data we come to know that one value would be negative and one value would be positive. Using the conditions 3 < x < 4 and – 9 < y < – 8 , the value of y would be negative and the value of x would be positive. Let us analyze the options: We can see that since y has been squared in option a, its value becomes positive. The value of xy2 would be positive and will not be the minimum. From (b) and (c), x2y and 5xy would give negative values but we cannot find the specific negative and positive values On comparing (a) and (c), we find that x 2 < 5x in 2 < x < 3. Since the value of y is negative therefore x 2y > 5xy
Therefore 5xy would give the minimum value.
Incorrect
Solution (c)
From the given data we come to know that one value would be negative and one value would be positive. Using the conditions 3 < x < 4 and – 9 < y < – 8 , the value of y would be negative and the value of x would be positive. Let us analyze the options: We can see that since y has been squared in option a, its value becomes positive. The value of xy2 would be positive and will not be the minimum. From (b) and (c), x2y and 5xy would give negative values but we cannot find the specific negative and positive values On comparing (a) and (c), we find that x 2 < 5x in 2 < x < 3. Since the value of y is negative therefore x 2y > 5xy
Therefore 5xy would give the minimum value.
-
Question 33 of 35
33. Question
2/5 of the voters promised to vote for P and the rest promised to vote for Q. Of these, on the last day 15% of the voters went back of their promise to vote for P and 25% of the voters went back of their promise to vote for Q, hence P lost by 4 votes. Then the total number of voters actually voting for P is
Correct
Solution (c)
Let the total number of votes = 100
2/5 of 100 = 40
Initially, there were 40 of these, who promised to vote for P.
Number of people who vote for Q = 60
On the last day, Number of people who broke their promise, to vote for P
= (15% of 40) = 6 voters
Number of people who broke their promise, to vote for Q = (25% of 60)
= 15 voters
So the number of votes P have = (40 – 6 + 15) = 49 votes
The number of votes Q have = (60 – 15 + 6) = 51 votes.
Because margin of victory for Q = (51 – 49) = 2
2% of total = 4
Hence the total number of voters is 200. The total number of voters for P is 49 × 2 = 98
Incorrect
Solution (c)
Let the total number of votes = 100
2/5 of 100 = 40
Initially, there were 40 of these, who promised to vote for P.
Number of people who vote for Q = 60
On the last day, Number of people who broke their promise, to vote for P
= (15% of 40) = 6 voters
Number of people who broke their promise, to vote for Q = (25% of 60)
= 15 voters
So the number of votes P have = (40 – 6 + 15) = 49 votes
The number of votes Q have = (60 – 15 + 6) = 51 votes.
Because margin of victory for Q = (51 – 49) = 2
2% of total = 4
Hence the total number of voters is 200. The total number of voters for P is 49 × 2 = 98
-
Question 34 of 35
34. Question
If a 4 digit number is formed with digits 1, 2, 3 and 5. What is the probability that the number is divisible by 25, if repetition of digits is not allowed?
Correct
Solution (a)
Total number of four-digit numbers that can be formed = 4!.
If the number is divisible by 25, then the last two digits are 25.
So the first two digits can be arranged in 2! ways.
Hence, required probability 2! /4! = 1/12
Incorrect
Solution (a)
Total number of four-digit numbers that can be formed = 4!.
If the number is divisible by 25, then the last two digits are 25.
So the first two digits can be arranged in 2! ways.
Hence, required probability 2! /4! = 1/12
-
Question 35 of 35
35. Question
One of the two digits of a two-digit number is three times the other digit. If you interchange the digits of this two-digit number and add the resulting number to the original number, you get 88. What is the original number?
Correct
Solution (c)
Let the unit place digit of a two-digit number be x.
Therefore, the tens place digit = 3x
∵ 2-digit number = 10 x tens place digit + unit place digit
∴ Original number = 10 * 3x + x=30x+x=31×10×3x+x=30x+x=31x
According to the question, New number + Original number = 88
=> 10x+3x+31x=88⇒10x+3x+31x=88
44x=88⇒ 44x=88
X=(88/44)=2
Hence, the 2-digit number = 31x=31*2=62
Incorrect
Solution (c)
Let the unit place digit of a two-digit number be x.
Therefore, the tens place digit = 3x
∵ 2-digit number = 10 x tens place digit + unit place digit
∴ Original number = 10 * 3x + x=30x+x=31×10×3x+x=30x+x=31x
According to the question, New number + Original number = 88
=> 10x+3x+31x=88⇒10x+3x+31x=88
44x=88⇒ 44x=88
X=(88/44)=2
Hence, the 2-digit number = 31x=31*2=62
All the Best
IASbaba